数论 Number Theory II|MATH 8822Boston College Assignment

0

Assignment-daixieTM为您提供波士顿学院Boston College MATH 8822 Number Theory II数论代写代考辅导服务!





Instructions:

Factorization of Ideals: The factorization of ideals in algebraic number theory is an extension of prime factorization in the integers. Given an algebraic number field K, we can define an ideal I to be a subset of K consisting of elements that are divisible by a fixed non-zero element of K. The factorization of an ideal into a product of prime ideals is unique up to order and multiplication by units of the ring of integers of K. This factorization has important applications in algebraic number theory, including the proof of Fermat’s Last Theorem.

Local Fields: A local field is a field that is complete with respect to a non-archimedean absolute value. Examples of local fields include p-adic numbers and function fields over a finite field. Local fields have important applications in number theory, representation theory, and algebraic geometry.

数论 Number Theory II|MATH 8822Boston College Assignment

问题 1.

Let $(L, \leq)$ be a partially ordered set in which every countable chain has an upper bound. Let $S \subseteq L$ be countable such that for all $a, b \in S$ there is $c \in S$ such that $a \leq c$ and $b \leq c$. Prove that $S$ has an upper bound in $L$.

证明 .

To prove that $S$ has an upper bound in $L$, we will use Zorn’s lemma. Let $T$ be the set of all chains in $L$ that are contained in $S$, and let $C$ be a chain in $T$. We want to show that $C$ has an upper bound in $L$.

Since $C$ is a chain in $L$, we know that $(C, \leq)$ is a partially ordered set. Let $c$ be an upper bound for $C$ in $L$. We claim that $c$ is also an upper bound for $S$.

Suppose for the sake of contradiction that $c$ is not an upper bound for $S$. Then there exists some $s \in S$ such that $s \nleq c$. Since $S \subseteq C$, we know that $s \in C$. Since $c$ is an upper bound for $C$, we have $s \leq c$. This contradicts the assumption that $s \nleq c$, so our claim is proven.

Therefore, $c$ is an upper bound for $S$. To complete the proof, we need to show that $c$ is in fact an element of $L$. Since every countable chain in $L$ has an upper bound, we know that $L$ is a complete partial order. In particular, every chain in $L$ has a least upper bound (lub).

Let $D$ be the set of all upper bounds of $S$ in $L$. We want to show that $c = \operatorname{lub}(D)$, which will complete the proof. First, we show that $c$ is an upper bound of $S$. We have already shown this above.

Now, let $d$ be an upper bound of $S$ in $L$. We want to show that $c \leq d$. By definition of $D$, we have $c \in D$, so $c$ is an upper bound of $S$. Therefore, we have $c \leq d$, since $d$ is also an upper bound of $S$.

Finally, we need to show that $c$ is the least upper bound of $D$. Suppose for the sake of contradiction that there exists some $d’ \in L$ such that $d’ < c$ and $d’$ is an upper bound of $D$. Since $c$ is an upper bound of $D$, we have $d’ \leq c$. But this contradicts the assumption that $c$ is the least upper bound of $D$. Therefore, $c$ must be the least upper bound of $D$.

We have now shown that $S$ has an upper bound in $L$. Therefore, by definition of a complete partial order, $L$ has a least upper bound for $S$, which completes the proof.

问题 2. Prove that for all nonnegative integers $n \geq 0$ and $k>1, \sqrt[k]{n}$ is rational if and only if $n=d^k$ for some integer $d$.

证明 .

First, let’s assume that $\sqrt[k]{n}$ is rational. This means that there exists some integers $a$ and $b$ such that $\sqrt[k]{n}=\frac{a}{b}$, where $b\neq 0$. Raising both sides to the $k$-th power gives us $n=\frac{a^k}{b^k}$, which can be written as $nb^k=a^k$. This shows that $a^k$ is a multiple of $b^k$, which implies that $a$ must be a multiple of $b$. Let $a=cb$ for some integer $c$. Then we have $n=cb^k$, which can be written as $n=(b^{k-1}c)^k$. Therefore, $n$ is of the form $d^k$ for some integer $d=b^{k-1}c$.

Now let’s assume that $n=d^k$ for some integer $d$. We can write $d$ as $d=be$, where $b$ is the largest integer such that $b^k$ divides $d^k$, and $e$ is some integer. Then $n=d^k=(be)^k=b^ke^k$. Since $b$ is the largest integer such that $b^k$ divides $d^k$, $e$ is not divisible by $b$. Therefore, $b$ and $e$ are relatively prime. This implies that $b^k$ and $e^k$ are also relatively prime. Thus, $n=b^ke^k$ is in its simplest form as a product of relatively prime integers, and its $k$-th root is $\sqrt[k]{n}=e\in \mathbb{Z}$. Therefore, $\sqrt[k]{n}$ is rational.

Combining the two directions of the proof, we have shown that for all nonnegative integers $n \geq 0$ and $k>1$, $\sqrt[k]{n}$ is rational if and only if $n=d^k$ for some integer $d$.

问题 3.

Suppose $p, q, r$ are integers such that $\sqrt{p}+\sqrt{q}+\sqrt{r}=s \in \mathbb{Q}$. Prove that $p, q, r$ are perfect squares.

证明 .

Let $s=\sqrt{p}+\sqrt{q}+\sqrt{r}$.

Squaring both sides, we get:

$$s^2=p+q+r+2\sqrt{pq}+2\sqrt{qr}+2\sqrt{rp}$$

Since $s$ is rational, $s^2$ is also rational. Thus, $\sqrt{pq}$, $\sqrt{qr}$, and $\sqrt{rp}$ must be rational.

Suppose, for contradiction, that $p$ is not a perfect square. Then, $\sqrt{p}$ is irrational, and $\sqrt{q}+\sqrt{r}=s-\sqrt{p}$ is also irrational.

But this implies that $\sqrt{pq}$ and $\sqrt{qr}$ are irrational, which contradicts the fact that they are rational. Thus, $p$ must be a perfect square.

Similarly, we can show that $q$ and $r$ are also perfect squares. Thus, $p$, $q$, and $r$ are all perfect squares, as required.

这是一份2023年的波士顿学院Boston College MATH 8822数论代写的成功案例




















数论 Number Theory I|MATH 8821Boston College Assignment

0

Assignment-daixieTM为您提供波士顿学院Boston College MATH 8821 Number Theory I数论代写代考辅导服务!





Instructions:

Number theory is a branch of mathematics that deals with the study of integers and their properties, relationships, and patterns. It includes topics such as prime numbers, divisibility, congruences, Diophantine equations, and the distribution of prime numbers.

Number theory is considered one of the oldest and most fundamental branches of mathematics. Its origins can be traced back to ancient civilizations such as the Babylonians, Greeks, and Indians. It has played a significant role in the development of many other areas of mathematics, including algebra, geometry, and analysis.

Some famous problems in number theory that have been solved include Fermat’s Last Theorem, which states that no three positive integers a, b, and c can satisfy the equation a^n + b^n = c^n for any integer value of n greater than 2, and the Four Color Theorem, which states that any map can be colored using only four colors such that no two adjacent regions have the same color.

Number theory also has many practical applications, including cryptography, coding theory, and computer science.

数论 Number Theory I|MATH 8821Boston College Assignment

问题 1.

(a) Let $S \subseteq\{1,2, \ldots, 2 n\}$ such that $|S| \geq n+1$. Prove that there exist $a, b \in S$ such that $a$ divides $b$.

证明 .

We can prove the statement using contradiction. Assume that there does not exist $a,b\in S$ such that $a$ divides $b$. This means that for any two distinct elements $a,b\in S$, $a$ does not divide $b$.

Let us consider the set of all numbers of the form $2^k$ where $1\leq k\leq n$. Since $|S|\geq n+1$, there must exist at least $n+1$ elements in $S$. By the Pigeonhole Principle, at least two of these elements, say $s_1$ and $s_2$, belong to the same subset of the form ${2^k, 2^{k+1}, \ldots, 2^{k+1}-1}$ for some $1\leq k\leq n-1$. Without loss of generality, assume that $s_1$ is the smaller of the two elements, i.e., $s_1< s_2$.

Now, since $s_1$ and $s_2$ belong to the same subset, they must have the same highest power of $2$ in their prime factorization. Let this power be $2^m$ where $m\leq n-1$. Therefore, we can write $s_1=2^m p_1$ and $s_2=2^m p_2$ where $p_1$ and $p_2$ are odd integers. Since $s_1<s_2$, we have $p_1<p_2$.

Now, consider the element $s_2/p_1$. Since $p_1<p_2$, we have $s_2/p_1>s_2/p_2=s_1$. Moreover, $s_2/p_1$ is an integer since $p_1$ divides $s_2$. Therefore, we have $s_1<s_2/p_1<s_2$ and $s_2/p_1\in S$.

However, we also have $s_2/p_1=2^m p_2/p_1$. Since $p_1$ is odd, $p_2/p_1$ is also odd. Therefore, $s_2/p_1$ is a multiple of $2^m$, and hence is in the same subset as $s_1$ and $s_2$. But this contradicts our assumption that $a$ does not divide $b$ for any two distinct elements $a,b\in S$. Therefore, our assumption must be false, and there exist $a,b\in S$ such that $a$ divides $b$.

问题 2. (b) Fix $r \geq 1$. Suppose $S \subseteq\left{1, \ldots, 2^r n\right}$ such that $|S|=\left(2^r-1\right) n+1$. Prove that there are $a_0<a_1<\ldots<a_r$ in $S$ such that $a_i$ divides $a_{i+1}$ for all $0 \leq i<r$.

证明 .

We will use induction on $n$ to prove the statement. For $n=1$, $S$ has only one element and the statement is trivially true.

Suppose the statement is true for $n-1$, and let $S$ be a subset of ${1,2,\dots,2^rn}$ of size $(2^r-1)n+1$. Let $T_1 = S \cap {1,2,\dots,n}$ and $T_2 = S \cap {n+1,n+2,\dots,2^rn}$. Note that $|T_1|+|T_2| = |S| \geq (2^r-1)n+1$, so either $|T_1| \geq (2^{r-1}-1)n+1$ or $|T_2| \geq (2^{r-1}-1)n+1$.

Without loss of generality, assume that $|T_1| \geq (2^{r-1}-1)n+1$. By the induction hypothesis, there exist $a_0 < a_1 < \dots < a_r$ in $T_1$ such that $a_i$ divides $a_{i+1}$ for all $0 \leq i < r$.

Now we consider the set $S’ = {a_i2^r : 0 \leq i \leq r}$. Since $|S’|=r+1$, there exist distinct $i,j$ such that $a_i2^r$ and $a_j2^r$ belong to $S’ \cap T_2$. Without loss of generality, assume that $a_i2^r < a_j2^r$ and $a_i2^r \in T_2$ and $a_j2^r \in T_2$. Then $a_i2^{r-1} \in T_1$, so we can add $a_i2^{r-1}$ to our sequence $a_0 < a_1 < \dots < a_r$ to obtain a new sequence $a_0 < a_1 < \dots < a_r < a_i2^{r-1}$ in $S$ with the desired divisibility property. This completes the induction step and the proof.


问题 3.

Suppose $k, n$ are integers. Prove that
$$
(k+\sqrt{n})^t+(k-\sqrt{n})^t
$$
is an integer for all $t \in \mathbb{N}$

证明 .

We will prove this by induction on $t$.

For $t=1$, we have $(k+\sqrt{n})^1+(k-\sqrt{n})^1 = 2k$, which is an integer.

Assume that for some $t \geq 1$, $(k+\sqrt{n})^t+(k-\sqrt{n})^t$ is an integer.

Then, using the binomial theorem, we have: $$(k+\sqrt{n})^{t+1}+(k-\sqrt{n})^{t+1} = (k+\sqrt{n})^t(k+\sqrt{n})+(k-\sqrt{n})^t(k-\sqrt{n})$$ Expanding both terms, we get: $$(k+\sqrt{n})^{t+1}+(k-\sqrt{n})^{t+1} = ((k+\sqrt{n})^t+(k-\sqrt{n})^t)k + \sqrt{n}((k+\sqrt{n})^t-(k-\sqrt{n})^t)$$

Since $(k+\sqrt{n})^t+(k-\sqrt{n})^t$ is an integer by the induction hypothesis, it suffices to show that $\sqrt{n}((k+\sqrt{n})^t-(k-\sqrt{n})^t)$ is also an integer.

To see this, we can use the fact that $(k+\sqrt{n})^t$ and $(k-\sqrt{n})^t$ are both solutions to the recurrence relation: $$a_{t+1} = 2ka_t – a_{t-1}$$ with initial conditions $a_0=2$ and $a_1=2k$.

Therefore, $\sqrt{n}((k+\sqrt{n})^t-(k-\sqrt{n})^t)$ is also a solution to this recurrence relation with the same initial conditions, and hence is an integer.

Thus, $(k+\sqrt{n})^{t+1}+(k-\sqrt{n})^{t+1}$ is an integer as the sum of two integers. By induction, the statement holds for all $t\in\mathbb{N}$.

这是一份2023年的波士顿学院Boston College MATH 8821数论代写的成功案例




















数论代写 Number Theory|MATH 305S Duke University Assignment

0

Assignment-daixieTM为您提供杜克大学Duke UniversityMATH 305S Number Theory数论学代写代考辅导服务!





Instructions:

Divisibility properties of integers, prime numbers, congruences, quadratic reciprocity, number-theoretic functions, simple continued fractions, rational approximations; contributions of Fermat, Euler, and Gauss. Prerequisite: Mathematics 122, 112L, 122L, or consent of instructor. Individual research paper required.

数论代写 Number Theory|MATH 305S Duke University Assignment

问题 1.

Let $a>0$ and $b$ be integers. Show that there is an integer $k$ such that $b+k a>0$. (Hint: use well-ordering.)

证明 .

Suppose not. Then let $S$ be the set of integers ${-(b+k a): k \in \mathbb{Z}}$, so by hypothesis $S$ consists entirely of nonnegative integers. By the Well-Ordering Principle, it has a smallest positive element, say, $b+k a$. But then $b+(k-1) a$ is smaller since $a>0$, contradiction.

问题 2.

Let $a$ and $b$ be positive integers whose ged is 1 . Find the largest positive integer $n(a, b)$ which is not a non-negative integer linear combination of $a$ and $b$. Prove your answer (i.e. show that $n(a, b)$ cannot be represented as $a x+b y$ with $x, y \in \mathbb{N} \cup{0}$ and that every greater integer can be represented in such a way).

证明 .

The largest such integer is $a b-a-b$. To see it’s not a nonnegative integer linear combination, suppose $a b-a-b=a x+b y$ with $x, y \in \mathbb{Z}_{\geq 0}$. Then $a(b-1-x)=b(y+1)$. And since $(a, b)=1$ we have $a \mid y+1$ (and $b \mid b-1-x$ ). This forces $y \geq a-1$ because $y+1 \geq 1$. So
$$
a x+b y \geq a \cdot 0+b(a-1)=a b-b>a b-a-b
$$
contradicting $a b-a-b=a x+b y$.
On the other hand, suppose $n>a b-a-b$. Since $\operatorname{gcd}(a, b)=1$ we can write $n=a x+b y$ with $x, y \in \mathbb{Z}$ (not necessarily nonnegative). Now note that $n=a(x-b k)+b(y+a k)$ for any integer $k$. By the division algorithm, there exists an integer $k$ such that $0 \leq x-b k<b$. Let $x^{\prime}=x-b k$ and $y^{\prime}=y+a k$. Then we have $n=a x^{\prime}+b y^{\prime}$ with $0 \leq x^{\prime} \leq b-1$, so
$$
b y^{\prime}=n-a x^{\prime} \geq(a b-a-b+1)-a(b-1)=-(b-1) .
$$
Therefore $y^{\prime} \geq \frac{-(b-1)}{b}$, and since $y^{\prime}$ is an integer, we get $y^{\prime} \geq 0$. This shows that $n=a x^{\prime}+b y^{\prime}$ is a nonnegative integer linear combination.

问题 3. Let $a>1$ be a positive integer, and $m, n$ be natural numbers. Show that $a^m-1 \mid a^n-1$ if and only if $m \mid n$. Show that the ged of $a^m-1$ and $a^n-1$ is $a^{(m, n)}-1$.

证明 .

One direction is clear: if $m \mid n$ then $n=m k$ for some positive integer $k$, and
$$
a^n-1=a^{m k}-1=\left(a^m-1\right)\left(a^{m(k-1)}+a^{m(k-2)}+\cdots+a^m+1\right)
$$
is divisible by $a^m-1$. Now if $m \nmid n$, we write $n=m k+r$ with $0<r<m$. Then
$$
a^n-1=a^{m k+r}-1=a^{m k+r}-a^r+a^r-1=a^r\left(a^{m k}-1\right)+a^r-1
$$
Now $a^m-1$ divides $a^{m k}-1$ but it doesn’t divide $a^r-1$, since $0<a^r-1<a^m-1$. So $a^m-1$ can’t divide $a^n-1$


这是一份2023年的杜克大学Duke University MATH 305S数论代写的成功案例




















数论代写 Number Theory|MATH 17500 University of Chicago Assignment

0

Assignment-daixieTM为您提供芝加哥大学University of ChicagoMATH 17500 Number Theory数论学代写代考辅导服务!





Instructions:

The study of primes is also a central topic in number theory. Primes are the building blocks of the integers, and understanding their distribution is a key area of research. One famous result in this area is the prime number theorem, which gives an asymptotic estimate for the number of primes less than a given number.

Congruences are another important tool in number theory. They allow us to work with remainders modulo a given integer, and they have many applications, such as in cryptography and coding theory.

Primitive roots are special integers that have interesting properties modulo a given integer. They are related to the multiplicative group of integers modulo that integer, and their existence is a deep result in number theory.

.

数论代写 Number Theory|MATH 17500 Duke University Assignment

问题 1.

Let $a$ and $b$ be positive integers whose gcd is 1 . Find the largest positive integer $n(a, b)$ which is not a non-negative integer linear combination of $a$ and $b$. Prove your answer (i.e. show that $n(a, b)$ cannot be represented as $a x+b y$ with $x, y \in \mathbb{N} \cup{0}$ and that every greater integer can be represented in such a way).

证明 .

The largest such integer is $a b-a-b$. To see it’s not a nonnegative integer linear combination, suppose $a b-a-b=a x+b y$ with $x, y \in \mathbb{Z}_{\geq 0}$. Then $a(b-1-x)=b(y+1)$. And since $(a, b)=1$ we have $a \mid y+1$ (and $b \mid b-1-x$ ). This forces $y \geq a-1$ because $y+1 \geq 1$. So
$$
a x+b y \geq a \cdot 0+b(a-1)=a b-b>a b-a-b
$$
contradicting $a b-a-b=a x+b y$.
On the other hand, suppose $n>a b-a-b$. Since $\operatorname{gcd}(a, b)=1$ we can write $n=a x+b y$ with $x, y \in \mathbb{Z}$ (not necessarily nonnegative). Now note that $n=a(x-b k)+b(y+a k)$ for any integer $k$. By the division algorithm, there exists an integer $k$ such that $0 \leq x-b k<b$. Let $x^{\prime}=x-b k$ and $y^{\prime}=y+a k$. Then we have $n=a x^{\prime}+b y^{\prime}$ with $0 \leq x^{\prime} \leq b-1$, so
$$
b y^{\prime}=n-a x^{\prime} \geq(a b-a-b+1)-a(b-1)=-(b-1)
$$
Therefore $y^{\prime} \geq \frac{-(b-1)}{b}$, and since $y^{\prime}$ is an integer, we get $y^{\prime} \geq 0$. This shows that $n=a x^{\prime}+b y^{\prime}$ is a nonnegative integer linear combination.

问题 2.

Let $n>1$ be a positive integer. Show that the polynomial identity
$$
(x-a)^n \equiv x^n-a \quad(\bmod n)
$$
holds for every integer $a$ if and only if $n$ is prime.

证明 .

Suppose $n$ is prime. Then, since the binomial coefficients in the middle vanish mod $p$,
$$
\begin{aligned}
(x-a)^n & \equiv x^n+(-a)^n \
& \equiv x^n+(-a) \quad(\bmod n) .
\end{aligned}
$$
Now for the converse. The polynomial congruence in particular means that $n$ must divide $\left(\begin{array}{l}n \ i\end{array}\right)$ for $i=1, \ldots, n-1$. We’ll see first that this implies $n$ must be a power of a prime.

Let $p$ be any prime dividing $n$. If $n$ is not a power of $p$, then the base $p$ expansion of $n$ does not look like 1 followed by a bunch of zeroes, so it’s either $n_r 0 \cdots 0$ with $n \geq 2$, or $n_r n_{r-1} \cdots n_i \cdots n_0$ with some $n_i \geq 1$ for $i<r$. In any case, let $k$ have the base $p$ expansion $10 \cdots 0$ (i.e., $k=p^r$ ). Then subtracting $k$ from $n$ in base $p$ doesn’t involve any carries, so $p \nmid\left(\begin{array}{l}n \ k\end{array}\right)$ and therefore $n \nmid\left(\begin{array}{l}n \ k\end{array}\right)$, contradiction. So $n$ must be a power of $p$.

Let’s assume $n$ is not a prime, so we now have $n=p^r$ with $r \geq 2$. Then it’s clear that subtracting $p^{r-1}$ (whose base $p$ expansion is $010 \cdots 0$ ) from $n$ in base $p$ will involve only one carry. So $p |\left(\begin{array}{c}r \ p^{r-1}\end{array}\right)$, and thus $n=p^r$ cannot divide this binomial coefficient, contradiction. Therefore, $n$ is indeed a prime.

问题 3. Let $p$ be a prime, and $e \geq 1$. Find all the solutions of $x^2 \equiv 1\left(\bmod p^e\right)$.

证明 .

We have.
$$
p^e \mid\left(x^2-1\right)=(x-1)(x+1)
$$
Suppose $p$ is odd. Then $p$ can’t divide both $x+1$ and $x-1$, since their difference 2 isn’t divisible by $p$, so $\left(p^e, x+1\right)=1$ or $\left(p^e, x-1\right)=1$. Hence $p^e \mid x-1$ or $p^e \mid x+1$, and the only two solutions are $x \equiv \pm 1$ $\left(\bmod p^3\right)$
Now suppose $p=2$. Then $x^2 \equiv 1\left(\bmod 2^c\right)$ means $x$ must be odd, so let $x=2 y+1$. We have
$$
2^e \mid(x-1)(x+1)=4 y(y+1)
$$
Note that if $p=2$ then $x=1$, and if $p=4$ then $x=1,3$. So let’s assume $e \geq 3$. Since $y$ and $y+1$ are obviously coprime, we have $2^{e-2} \mid y$ or $2^{e-2} \mid y+1$, i.e., $y \equiv 0\left(\bmod 2^{\varepsilon-2}\right)$ or $y \equiv-1\left(\bmod 2^{c-2}\right)$. Then, modulo $2^{c-1}$, the possible solutions for $y$ are $0,2^{c-2}, 2^{e-2}-1,-1$, and the corresponding solutions for $x$ are $1,-1,2^{e-1}+1,2^{e-1}-1$. It’s easy to verify that all of these work and are distinct modulo $2^e$.


这是一份2023年的芝加哥大学University of Chicago MATH 17500数论代写的成功案例